IB Physics: Thin Film Interference

Поделиться
HTML-код
  • Опубликовано: 6 авг 2024
  • Examines the interference patterns found on thin films.
    Still 150+ free and 60+ exclusive videos. 10% of proceeds to charity. For IA mentoring or Online tutoring, visit donerphysics.com.
    IB Physics Topic 9.3
    Click JOIN to become a member of C. Doner’s IB Physics RUclips Channel, or become a patron by visiting / cdoner .
    Level/Tier 1: $1.99 per month gives you access to a google sheet with a full updated, well organized, linked listing of all the videos and other resources such as notes, worksheets and solution sets.
    Level/Tier 2: $4.99 per month gives you access to new videos. The new videos include IB style multiple choice and problem solving questions with full explanations, previews/reviews of the course content, and updates to the video lessons.
    After becoming a member or patron, go to bit.ly/donervidlist and request access to the video listing.
    "I would just like to say a massive thank you to you for uploading your videos. I started the IB [and]decided to try HL Physics as it seemed interesting. It turned out to be very far from my forté and I was consistently getting 3s and 4s in tests and mock exams. So a few months before the exam period of this year, I began to panic...Then I discovered your channel. For four weeks prior to paper one, I virtually worshipped your videos, and worked along to them in my 'Big Book of Revision.' Last week, I received my results. Incredibly and almost unbelievably, I obtained a 6 in HL Physics. Not only that, but I was 3 marks off a 7."
    Table of Contents:
    0:00 Introduction
    00:02 - Examples of Thin Films
    00:44 - What causes thin films?
    02:38 - Two contributions to the path difference / phase difference
    13:54 - Deriving Thin Film Formulas
    17:50 - IB Physics Data Booklet
    18:37 - Thin Film Interference Problem 1
    21:08 - This Film Interference Problem 2
    23:41 - Summary

Комментарии • 64

  • @MB-zo2xw
    @MB-zo2xw 5 лет назад +8

    you're such a god, you explain things that IB expects you to know but are never mentioned in their books

  • @greengoku10
    @greengoku10 7 лет назад +22

    Hey, I just wanted to thank you for these videos. You're a brilliant teacher and make my love for physics even greater. I sincerely hope your viewership increases exponentially, because more people need these videos. So thanks again

  • @yan_man23
    @yan_man23 6 лет назад +74

    Did you mean 0 or 2 inversions at 18:21? Thanks for the videos by the way.

    • @donerphysics
      @donerphysics  6 лет назад +43

      Thanks...should be 0 or 2 inversions.

    • @aravsriagarwal4056
      @aravsriagarwal4056 3 года назад +4

      @@donerphysics Thanks for the clarification!

  • @ananyabeura7224
    @ananyabeura7224 4 года назад +8

    Thank you so much for this. i don't know why the IB expects you to know this without even explaining or mentioning it in the text books

    • @donerphysics
      @donerphysics  4 года назад +26

      It is thin (some pun intended.)

  • @taliyaloz8878
    @taliyaloz8878 6 лет назад

    Great content! Thank you!!

  • @raghavcherukuru4936
    @raghavcherukuru4936 6 лет назад +8

    Legend, this helped me a lot.

  • @noenicolas2815
    @noenicolas2815 3 года назад +1

    Exactly what I needed. Perfect explanation

  • @VALR1able
    @VALR1able 4 года назад +1

    This video was very helpful. Thank you very much.

  • @vanessawertheim
    @vanessawertheim 2 года назад

    Thank you so much for this helpful video!

  • @ediewhittington3255
    @ediewhittington3255 5 месяцев назад

    Thank you! Really well explained!

  • @neet5080
    @neet5080 10 месяцев назад

    Great explanation. I was so amazed.

  • @sachinprabhuram5765
    @sachinprabhuram5765 3 года назад

    I did a question on N19 that says that when there are two inversions like in 13:24 there is a path difference of 2d. How? Question is Q10 b) (ii) in N19 for reference

  • @ashishsehrawat9091
    @ashishsehrawat9091 4 года назад +2

    You are awesome sir

    • @donerphysics
      @donerphysics  3 года назад +2

      Thank you. Please spread the word about the channel, become a subscriber or a member.

  • @longtranhai2761
    @longtranhai2761 5 лет назад +2

    The plus a 0.5 for 1 inversion is not included in the formula. Do we just add it ourselves if there is one inversion and in that case will it not be +lambda/2? Thank you for the videos.

    • @niya.s.
      @niya.s. Год назад +1

      You probably don't care anymore but no you don't need to add the 1/2 for 1 inversion cus it's already included in the formula. In the case of the destructive interference the 1/2 cancels out and then multiplies by lambda on both sides, which is why the IB formula says 2dn=m*lambda. And for constructive interference, the IB formula subtracts by 1/2 on both sides then multiplies by lambda on both sides which is why 2dn = (m + 1/2)*lambda.
      And, Mr. Doner said to swap the equations for 0 or 2 inversions which makes sense since it'd be the equations if 1/2 wasn't add to 2dn/lambda.

  • @ezu8501
    @ezu8501 2 года назад

    YOURE A GOD

  • @abhishekpillai5207
    @abhishekpillai5207 Год назад +1

    Hi! At the question in 19:59, how come the second ray isn't inverted when coming back out from the methyl iodide film to the glass?

    • @donerphysics
      @donerphysics  7 месяцев назад +1

      There is no phase change because the index of refraction of the second material is larger than that of the final material.

  • @andreshiguematu9207
    @andreshiguematu9207 4 года назад

    Souldn't you divide the index of refraction of the film by the index of refraction of the surroundings in order to appropriately convert lamba from the film to lamba in the surroundings?

    • @donerphysics
      @donerphysics  4 года назад

      Yes, we must generally use the relative index of refraction. The example was the common case in which the first medium is air for which n=1.

  • @justalittlebitoflove6520
    @justalittlebitoflove6520 3 года назад

    In the first example, I considered the air as the first medium, and thus concluded two inversions. Why is air not considered in this case?

    • @donerphysics
      @donerphysics  3 года назад +1

      Good question. Most problems involve air. There are two ways to know. One would be by having experiences with this type of application (which is unlikely) , and the other would be to recognize that no thickness is given for the glass and therefore we can not treat it as a thin film, just as we do not treat the air as a thin film.

  • @ezu8501
    @ezu8501 2 года назад

    Hey, can you explain what the m represents? do you have any videos explaining?

    • @donerphysics
      @donerphysics  2 года назад

      It is explained thoroughly in previous videos.. We get constructive interference when the the path difference is an integer number of wavelengths. m represents that integer.

  • @jasonfong7394
    @jasonfong7394 Год назад

    Hi, sorry to bother you. I was wondering if you could break down the
    optically dense --> rare --> dense
    scenario for thin film interference. Thanks!

    • @donerphysics
      @donerphysics  Год назад

      most common is air-film-air where we get a single phase inversion. the other situation you will see is air- more dense-even more dense, where there are two phase inversions which is equivalent to none at all.

  • @letseat1098
    @letseat1098 2 года назад

    Sir, what are the factors affecting the thin film and how to protect the thin film?

    • @donerphysics
      @donerphysics  2 года назад

      What thin film are you referring to?

  • @sidninja9518
    @sidninja9518 3 года назад

    23:38 would the answer be 189nm because rearranging the equation gives us t=(1/2lambda)/2n? I’m confused how the denominator becomes 4n

    • @donerphysics
      @donerphysics  3 года назад

      There is a 2 on the LHS as well as the RHS. The one the LHS represents a half wavelength of path difference that is necessary for destructive interference. The one on the RHS represents twice the thickness since the reflected beams travel through the thickness twice.

  • @yewweeyak3226
    @yewweeyak3226 11 месяцев назад

    Shouldn't you account for the refractive index of glass as well?

    • @donerphysics
      @donerphysics  11 месяцев назад

      Neither of the interfering rays travel through the glass so no.

  • @zhenghongchen8365
    @zhenghongchen8365 2 года назад

    Mr. Doner, in question 1( 21:00), which answer is the correct answer or all the answers are the correct answer? Please further clarify a little bit 😂 thanks

    • @donerphysics
      @donerphysics  2 года назад

      At the thicknesses indicated the reflected rays will be in phase.

  • @spifuntastic621
    @spifuntastic621 Год назад

    20:34, didn't you say we have to convert the wavelength value according to the medium? So shouldn't we convert the wavelength from 500nm to that in methyl iodide?

  • @engila9295
    @engila9295 5 лет назад

    Hello! Thank you for the video but I have 2 questions:
    1. What does the m stand for in the equation?
    2. Why are the interference equations still applied for 0 or 2 inversions? I thought interference patterns only occured for 1 inversion.

    • @donerphysics
      @donerphysics  5 лет назад

      1. m can be any number 0,1,2,3 etc., so m is a counter. Waves produce destructive interference when they are out of phase by half of a wavelength, or 1 and a half wavelengths etc. 2. No, we get stable interference patterns when two waves of the same frequency pass through each other.

    • @engila9295
      @engila9295 5 лет назад

      Chris Doner but why was m=0 used when thin film’s thickness was assumed to be minimal? How would the phase difference and thickness denote the number for m?

    • @donerphysics
      @donerphysics  5 лет назад

      At what time in the video are you referring to?

    • @engila9295
      @engila9295 5 лет назад

      Chris Doner around 20:10

    • @donerphysics
      @donerphysics  5 лет назад

      @@engila9295 20:10 is the summary. Is there a specific case that you are referring to? I have already answered your question in general.

  • @tauhid9983
    @tauhid9983 5 лет назад

    @22.55 Hey Mr Doner, could you please explain why there are 2 inversions

    • @donerphysics
      @donerphysics  5 лет назад

      Any time the index of refraction increases we get an inversion. The index increases twice.

    • @tauhid9983
      @tauhid9983 5 лет назад

      @@donerphysics So if for example the index of refraction of the film is equal to the index of refraction of the medium through which light come out of the film, would we get one inversion?

    • @donerphysics
      @donerphysics  5 лет назад

      @@tauhid9983 If they truly have the same index, they are the same material and there is no reflection.

    • @tauhid9983
      @tauhid9983 5 лет назад

      @@donerphysics therefore no inversion... hence if there's reflection between mediums, like you've marked the 2 point of reflection on the video, there will be 2 inversions

  • @tauhid9983
    @tauhid9983 5 лет назад

    Hell Mr Donner, could you explan whether or not, with respect to thin films, will be an inversion for an air wedge

    • @donerphysics
      @donerphysics  5 лет назад +1

      The (somewhat) relevant reflections are glass to air (no inversion) and air to glass (an inverstion), however, for air wedges the inversion isn't important since you are usually just interested in the idea that everytime a new fringe is produced, the path difference (2t) has increased by one wavelength.

    • @tauhid9983
      @tauhid9983 5 лет назад

      @@donerphysics So in IB question... we can take any equation 2t=nλ/2 or 2t=nλ..... to find for example the thickness of the film

  • @aviratthakor8974
    @aviratthakor8974 3 года назад

    I am totally lost. Why at 10:35 do we need to multiply by the index of refraction of the film?

    • @aviratthakor8974
      @aviratthakor8974 3 года назад

      And why would there be constructive or destructive interference if there are 0 or 2 inversions (18:09)? Wouldn't the reflected rays be in the same phase?

    • @donerphysics
      @donerphysics  3 года назад

      The wavelength changes whenever the medium changes. The higher the index of refraction the shorter the wavelength. The path difference is created when the light travels through the film, so we must use the wavelength within the film.

    • @donerphysics
      @donerphysics  3 года назад

      @@aviratthakor8974 Good. O or 2 inversions has the same effect. To get the rays out of phase we need the light that travels through the film to travel an extra half wavelength or one and one half wavelengths etc.